解析力学③補足① ルジャンドル変換 このエントリーをはてなブックマークに追加

ルジャンドル変換とは?

数学的にラグランジアンとハミルトニアンの関係を考えてみます。

ラグランジアンからハミルトニアンの変換

たとえば、解析力学ではラグランジアン$L$からハミルトニアン$H$への変換を行います。これは実はルジャンドル変換になっています。いま、実はオイラー・ラグランジュ方程式を考えればわかることですが、ラグランジアンは$q$,$\dot{q}$の関数ですが、ハミルトニアンは$q$と共役運動量$p$の関数になっています。

もちろん変数変換以上の意味を持っていて、ルジャンドル変換と名前がついています。ラグランジアンの全微分を考えます。
\begin{align*} dL=\sum_{i}\left(\dfrac{\partial L}{\partial q_i}dq_i+\dfrac{\partial L}{\partial \dot{q}_i}d\dot{q}_i\right) \end{align*}
ここで、共役運動量の定義を用いれば、
\begin{align*} dL=\sum_i \left(\dfrac{\partial L}{\partial q_i}dq_i+p_id\dot{q}_i\right) \end{align*}
いま、作りたいのは$\dot{q}_i$から$p_i$に変数を変えた関数ですね。つまり、上の微小量を考えるうえで$d\dot{q}_i$が邪魔です。この文字を消すために、以下のようにハミルトニアンを定義することにします。
\begin{align*} H&=\sum_i \dot{q}_ip_i-L \\ \end{align*}
この微小量をとると、再び共役運動量の定義を用いて、
\begin{align*} dH&=\sum_i\left(p_i\ d\dot{q}_i+\dot{q}_i\ dp_i\right)-\sum_{i}\left(\dfrac{\partial L}{\partial q_i}dq_i+\dfrac{\partial L}{\partial \dot{q}_i}d\dot{q}_i\right) \\ &=\sum_i\left(p_i\ d\dot{q}_i+\dot{q}_i\ dp_i\right)-\sum_{i}\left(\dfrac{\partial L}{\partial q_i}dq_i+p_i\ d\dot{q}_i\right) \\ &=\sum_i \left(\dot{q}_i\ dp_i-\dfrac{\partial L}{\partial q_i}dq_i\right) \end{align*}
これで微小量$\dot{q}_i$がいなくなりました。ところで、ハミルトニアンはこの式から$p_i$と$q_i$の関数だとわかりますね。よって、以下のようにしてみましょう。
\begin{align*} dH=\sum_i \left(\dfrac{\partial H}{\partial p_i}dp_i+\dfrac{\partial H}{\partial q_i}dq_i\right) \end{align*}


1次元のルジャンドル変換を考える

1次元の場合にはハミルトニアンは以下のようになります。
\begin{align*} H=\dfrac{\partial L}{\partial \dot{q}}\dot{q}-L =p\dot{q}-L \label{eq:1} \end{align*}
式を少し変形して、
\begin{align} L&=\dfrac{\partial L}{\partial \dot{q}}\dot{q}-H \label{eq:2} \end{align}
逆に、$\dfrac{\partial H}{\partial p}=\dot{q}$なので、
\begin{align} L&=\dfrac{\partial H}{\partial p}p-H \\ \therefore H=&=\dfrac{\partial H}{\partial p}p-L \label{eq:3} \end{align}
というようにかなり対称的な式になっています。つまり、「変換」したのちに「元に戻す」という操作が許されそうです。

ところで、\eqref{eq:2}を考えると、$L-\dot{q}$グラフ上で、切片が$-H$の直線のように見えると思います。つまり、$\dot{q}$と$L$を指定すれば、$p$と$H$が決まります。逆に、\eqref{eq:3}では$p$と$H$が決まれば、$\dot{q}$と$L$が求められます。

ところで、もし、$\dot{q} \mapsto p$が単射でないなら、同じ値の運動量に対して、二つの$\dot{q}$が存在します。つまり、ラグランジアンがある$p$に対して複数存在する可能性があります。これは不適切ですね。つまり、$\dot{q}\mapsto p$は単射である必要があります。

$\dot{q}\mapsto p$は以下のようにあらわされたのでした。
\begin{align*} p=\dfrac{\partial L}{\partial \dot{q}} \end{align*}
これが単射であるためには単調関数であればよいでしょう。つまり、
\begin{align*} \dfrac{\partial p}{\partial \dot{q}}=\dfrac{\partial^2 L}{\partial \dot{q}^2} \end{align*}
の符号がずっと一定であればよいことになります。つまり、ラグランジアンは凸関数または凹関数である必要があります。同様のやり方でハミルトニアンが凸関数、または、凹関数である必要があります。

この条件が満たされていないと、ルジャンドル変換を2回やってももとに戻らないということになります。

ハミルトニアンは凸関数

ハミルトニアンは凸関数(下に凸な関数)である必要があります。ハミルトニアンはポテンシャルが速度に依存しないなら全エネルギーと同じになります。凹関数(上に凸)だとどこまでも低いエネルギーを取ってしまうことになり不適切です。



このエントリーをはてなブックマークに追加